LSAT and Law School Admissions Forum

Get expert LSAT preparation and law school admissions advice from PowerScore Test Preparation.

 Administrator
PowerScore Staff
  • PowerScore Staff
  • Posts: 8916
  • Joined: Feb 02, 2011
|
#24063
Complete Question Explanation

Weaken. The correct answer choice is (A)

In this science-based stimulus, we learn about the carbon dating of rock shelters used by humans. While the test results show that human activity began about 20,000 years ago, "skeptics" believe that the tests may be inaccurately dating "old carbon" that percolating water may have carried to the site.

The question stem asks for the answer choice which weakens the skeptics' argument. The correct answer choice will likely either rule out the skeptics' hypothesis, or strengthen the case that the carbon-dating test is accurate.

Answer Choice (A): This is the correct answer choice. If, as this choice provides, percolating water would have affected all levels equally, then this rules out the skeptics' explanation.

Answer Choice (B): The fact that there have been some unsuccessful carbon dating efforts has no effect on the argument in the stimulus.

Answer Choice (C): The stimulus doesn't deal with the use of coal for fuel—just with the use of carbon content to estimate age.

Answer Choice (D): This potentially confusing answer offers the following: when they reattempted the carbon dating, the test did not show the deeper layers to be older. This suggests that on the second try, the test was inaccurate. This would not weaken the skeptics' argument—if anything, it might strengthen it.

Answer Choice (E): Some site has to have the oldest carbon; this does not automatically reduce the credibility of carbon dating.
 rachue
  • Posts: 140
  • Joined: Jun 22, 2011
|
#1761
Hi,

I don't understand the reasoning given in the explanation for this problem for A. We know that the rocks are consistently-spaced, time-wise, according to human activities of each era. If all of the rocks were affected equally by the "old carbon", wouldn't that have pushed all of them backward or forward in time? In so doing, wouldn't that prevent us from having an accurate idea of when ANY of the rocks were from, thereby STRENGTHENING the skeptics' view?

I really am having a hard time understanding the logic in this one. I'd appreciate any help. There must be something crucial I'm missing but I just can't seem to figure it out.
 Steve Stein
PowerScore Staff
  • PowerScore Staff
  • Posts: 1153
  • Joined: Apr 11, 2011
|
#1774
In the stimulus the author points out that the sample findings formed a consistent series, beginning with the present. If answer choice A is true, then this kills the skeptics' claim that old carbon contaminated the findings--such contamination would have affected the top layer as well, so that top layer would not have dated to the present. Since we know that the top layer did date to the present, this choice would weaken the skepics' claim.
 rachue
  • Posts: 140
  • Joined: Jun 22, 2011
|
#1777
AHH, ok. I keep trying to question premises when I need to keep in my mind that they're fact. Thanks for clarifying that for me.
 lawschoolforme
  • Posts: 33
  • Joined: Oct 15, 2013
|
#12460
Hello,

On this problem, although my contenders ended up being A and D, I don't feel strongly that either choices argues against the suggestion of the skeptics. How does A weaken the argument, and how does D not weaken the argument?

Thanks!

-lawschoolforme
 David Boyle
PowerScore Staff
  • PowerScore Staff
  • Posts: 836
  • Joined: Jun 07, 2013
|
#12480
lawschoolforme wrote:Hello,

On this problem, although my contenders ended up being A and D, I don't feel strongly that either choices argues against the suggestion of the skeptics. How does A weaken the argument, and how does D not weaken the argument?

Thanks!

-lawschoolforme
Hello,

D may hurt the reliability of the samples, but it doesn't address the skeptics' thesis about "percolation" the way that A does. It might actually strengthen their argument in general, though.
A says that the top sample should have been infected too, but the rest of the stimulus says that the dates went in an orderly fashion, a consistent series, getting earlier the deeper they went.

David
 lawschoolforme
  • Posts: 33
  • Joined: Oct 15, 2013
|
#12727
Hi David,

Thanks for your explanation!

Er, I'm unfortunately still having a bit of trouble though :oops: . Could you clarify how D might actually strengthen a skeptics' argument?

I guess my confusion with this problems comes from the two scenarios I have playing in my head about this problem:

Scenario 1: Groundwater infects top layer the most, bottom layer the least

Scenario 2: Groundwater infects all layers equally with same amount of carbon

By saying that A is true, we're saying that if there was contamination, the orderly fashion would have been disrupted. But D is saying that the orderly system was not disrupted, so doesn't this mean that there wasn't any contamination?

Thanks so much again for all your help!

-lawschoolforme
 Lucas Moreau
PowerScore Staff
  • PowerScore Staff
  • Posts: 216
  • Joined: Dec 13, 2012
|
#12736
Hello, lawschoolforme,

The skeptics are suggesting that only the bottom layers have been contaminated by "old carbon", thus making them appear older than they actually are. If D is true, that doesn't at all disprove the argument of the skeptics - D is basically saying that every layer is older than the one above it. D could suggest that the skeptics' argument is correct because of how much older the bottom-most layers are than the top-most. It may strengthen, but does not necessarily do so.

However, if A is true, there's no way that the bottom layers could have been contaminated without contaminating the upper layers also. That shatters the skeptics' argument that the bottom layers were affected in a way that the top layers were not.

Hope that helps,
Lucas Moreau
PowerScore
 PositiveThinker
  • Posts: 49
  • Joined: Dec 24, 2016
|
#34855
I picked C but now i see how the answer is A. Just to clarify, is C wrong because it specifies "coal fuel?" Even though the stimulus only says coal deposits and nothing about fuel specifically? I only ask because this type of assumption could be on another question.

For example, say the stimulus had said "old carbon carried by percolating groundwater by near by coal deposits *used for fuel*"

would the reasoning in C been an acceptable answer?

thanks!
User avatar
 Jonathan Evans
PowerScore Staff
  • PowerScore Staff
  • Posts: 726
  • Joined: Jun 09, 2016
|
#35012
Hi, Positive Thinker,

Good question. Even if the stimulus had described "coal for fuel" answer choice (C) would have still had problems; to wit, the absence of evidence of use of fuel at the time does not indicate evidence of absence; just because there is no evidence that people were using fuel at the time doesn't mean they weren't, but now to borrow an old adage criticizing Scholasticism and medieval philosophy, we're debating how many angels can dance on the head of a pin.

For this problem and all LR problems, it's essential to stick to the precise meaning of the text. Given that we know that there is a "consistent series" in the rock sample dating going from the most recent samples to the earliest, answer choice (A) gives substantially stronger evidence against the author's conclusion, since the contamination would likely have thrown off the whole sequence.

I hope this helps!

Get the most out of your LSAT Prep Plus subscription.

Analyze and track your performance with our Testing and Analytics Package.